MONT Typos and Corrections: Aditya Khurmi
MONT Typos and Corrections: Aditya Khurmi
MONT Typos and Corrections: Aditya Khurmi
∗
Aditya Khurmi
Last Updated: June 8, 2021
There are minor typos, which include small things like grammatical errors or mathe-
matical errors which can be easily guessed as being typos. Then there are major typos
which are big typos that change the problem/statement. These are marked with !!!
All typos are in red color, while corrections (such as deletion of a problem) are written
normally in black. One common typo appearing a lot is changing ”.
P.g. 16 We finish this discussion with a list of properties, some of which we discussed above, and some
others, which I leave as exercises to prove.
P.g. 17 For 2 fixed integers x, y, prove that
x − y | xn − y n
for any positive integer n. (Hint: Long division)
P.g. 19 So, for instance, you can factorize 45 as 32 × 5 but not in any other way.
P.g. 19 Theorem 1.5.1 (Divisibility in Sets). Let a, b be two positive integers. Then
a | b ⇔ A⊆B.
1
1
Aditya Khurmi (Last Updated: June 8, 2021) MONT Typos and Corrections
P.g. 65
1 1 1
+ 2 + ≡ +(12 + 22 + · · · + (p − 1)2 )
12 2 (p − 1)2
(p − 1)p(2p − 1)
=+ (mod p),
6
P.g. 65 note where we use = and where ≡, denoting where we used algebraic facts vs number theoretic
facts
P.g. 66 (delete an extra “if”) So if we can show the sequence hai i eventually becomes constant . . .
P.g. 67 a + b + c ≥ 3c + 3d ≥ 3d.
P.g. 69 So this problem is screaming at us to try to do what he did, show that the “set” is infinite; in his
case the set of primes, and in our case the set M.
P.g. 71 Regional Mathematical Olympiad
P.g. 72 !!! every positive integer has at least as many divisors of the form 4k + 1 as divisors of the form
4k + 3
P.g. 75 !!! However, there are some technical details you need to know to fully appreciate the proof, so
you can find it in the special section of the chapter: Modular Arithmetic Advanced.
√
P.g. 79 Because of the 2 in 2 n, we feel some pairing type argument might be involved in the proof.
P.g. 79 Indeed, if d | n, then (n/d) | n. too.
P.g. 81 There are thus ϕ(m) numbers in each row coprime to m.
P.g. 82 Use the integral test to show that ζ(s) converges if and only if s > 1. In particular, show that ζ(1)
diverges.
P.g. 83 The product somehow balances out each others’ growth).
P.g. 84 This function is very useful because of the following 2 properties:
P.g. 85 We observe the sum with indices varying over d | n are a common theme in multiplicative functions.
P.g. 86 (Delete the 5th point “µ ∗ 1 = δ.”)
2
2
Aditya Khurmi (Last Updated: June 8, 2021) MONT Typos and Corrections
P.g. 89 Is this the same proof as the one we gave here: 3.3.1?
P.g. 89 (in the footnote) Technically this isn’t an Arithmetic Function since they are R → Z. However, I
still cover them in this chapter.
P.g. 89 The graph suggests why it is called the “floor function”
P.g. 90 For negative, it’s not exactly the decimal part, but its complementary.
P.g. 90 The definitions give the following properties
P.g. 90 Remove the PreRMO problem.
P.g. 95 (Delete “One useful lemma we obtain this way is”)
P.g. 96 So it suffices to show d(n + 1) > an and d(n + 1) < an both hold infinitely often (why?).
P.g. 96 (Latex error) Hence, d(n + 1) > max{d(1), . . . d(n)} ≥ an , as desired.
P.g. 96 !!! Here, the rows and columns are 1, 2, . . . , n, and an element (i, j) is 1 if i is a multiple of j,
and 0 otherwise.
P.g. 96 !!! Fix a column say the ith one. Then, the number of 1s here is ni .
P.g. 96 !!! Next, if we fix a row, say the ith one, then the number of 1s here is the number of divisors of
i (why?).
P.g. 97 !!! Now, suppose in our table, instead of writing 1s, we write the multiple, i.e. the (i, j) element
is j if i is a multiple of j, and 0 otherwise. So, the table for n = 8 now is
1 2 3 4 5 6 7 8
1 1 0 0 0 0 0 0 0
2 1 2 0 0 0 0 0 0
3 1 0 3 0 0 0 0 0
4 1 2 0 4 0 0 0 0
5 1 0 0 0 5 0 0 0
6 1 2 3 0 0 6 0 0
7 1 0 0 0 0 0 7 0
8 1 2 0 4 0 0 0 8
3
3
Aditya Khurmi (Last Updated: June 8, 2021) MONT Typos and Corrections
x0 → x1 → x2 → . . .
of numbers satisfying property P. The most common form of argument proceeds by showing
x0 > x1 , which then gives x1 > x2 and so on. If you can show that xi ∈ Z always, and that only
positive numbers satisfy P, then you have a contradiction. (why?)
There are many variants of the argument above, which basically induce an infinite chain and show
that this must be impossible. At heart, an infinite descent argument can be translated into an
argument involving the extremal principle (by saying pick the smallest x satisfying property P ),
however infinite descents are easier to motivate.
It is best understood by examples, so let’s take a look, starting with the IMO 1988 problem itself:
P.g. 118 Then, if we have a lattice point (x, y) ∈ H, then by Vieta the point (ky − x, y) is also a lattice
point on H. Further, we can show that ky − x < x and so the x coordinate is lower.
P.g. 118 In retrospect, we realize that we can shorten some of our work.
P.g. 118 If at this point, we have x > b, then we repeat everything we just did and get to (a, b) from (x, b),
since a is the other root of the quadratic x forms.
4
4
Aditya Khurmi (Last Updated: June 8, 2021) MONT Typos and Corrections
P.g. 119 Then, once we show x ≤ a is a positive integer, so that (x, b) is also a valid pair, we say that
x + b < a + b, which contradicts the minimality. From here on, we will use either descent or the
extremal principle to phrase our argument, depending on which one is easier.
P.g. 120 !!! (Delete part in red) holds unless (a, b) = (2, 1) (why?). If (a, b) = (2, 1), then k = 6/2 = 3, as
desired. So suppose not
P.g. 127 !!! In Problem 4.9.1, change the equation to
x2 y + y 2 z + z 2 x = 3xyz.
P.g. 133 At the end of the day, solving some sort of equation is one of the key goals of a mathematician.
That is what led them to discover Z, Q, R and C. This is what we have done in the last chapter on
modular arithmetic too. For instance, in solving the equation ax − b ≡ 0 (mod p), we were led to
the concept of inverses.
P.g. 133 One of the equations that led humanity to discover irrationals was x2 = 2.
P.g. 134 Now let’s consider the equation that led humans to discover the complex numbers: x2 = 1.
P.g. 134 So, x2 ≡ −1 (mod 5) has the solutions x ≡ 2, 3.
p−1 p−1
P.g. 135 x2 ≡ −1 (mod p) =⇒ (x2 ) 2 ≡ (−1) 2 (mod p).
P.g. 136 For instance, when p = 5, we have 1 ≡ 1, 42 ≡ 1 (mod 5).
2
P.g. 157 Let’s now present one property which is going to be themost important result related to νp :
5
5
Aditya Khurmi (Last Updated: June 8, 2021) MONT Typos and Corrections
b = pxn+y `, 1 ≤ y ≤ n − 1.
P.g. 166 !!! ( LTE for addition ) Let p > 2 be a prime and a, b ∈ Z be coprime to p such that p | a+b.
P.g. 167 “Clearly, a = 1 works since then 4(1n + 1) = 8 for all n, which is a cube. Now, we will show this
is the only possibility.” The proof we gave follows after this. Don’t miss this “obvious” statement
and lose marks!
P.g. 169 Example 6.6.1 (Paul Erdös)
P.g. 170 !!! If νp (x) = y, think of x as py , not as cpy .
P.g. 170 !!! In this case, if νp (ax + by + cz ) ≥ 0, we must have that νp (by ) = νp (cz )
P.g. 170 !!! Hence, yνp (b) = zνp (c). If gcd(y, z) = d and y = y ∗ d, z = z ∗ d, then we must have νp (b) = kz ∗
and νp (c) = ky ∗ for some k. Hence,
6
6
Aditya Khurmi (Last Updated: June 8, 2021) MONT Typos and Corrections
P.g. 185 !!! Theorem 7.1.8 (Fundamental Theorem of Algebra). Every polynomial of degree at least 3 in
R[X] is reducible into linear and quadratic factors in R[X].
P.g. 185 Now since at least one of c1 , c2 is greater than 1 (why?), hence there is a prime p dividing c1 c2 .
P.g. 185 !!! (Add the following line at the end)
Compare the coefficients of xn+m in (c1 c2 )f = (c1 g)(c2 h). On the left side, it is kn+m (which is
divisible by p), while on the right side it is an bm . So p | an or p | bm , say the former.
P.g. 186 !!! So pick the largest 0 ≤ i ≤ n and 0 ≤ j ≤ m such that p - ai , bj .
P.g. 186 So, identical polynomials means “exactly the same”, i.e. carbon copies. The important point in
the definition is that it says the coefficients are same, and says nothing about the values. We say
that they are f̧ormally” equal (“formal” in context of polynomial is used for coefficients)
P.g. 187 Each term serves as an “indicator term”
(x−2)(x−3)(x−4) (x−3)(x−4)(x−1) (x−4)(x−1)(x−2) (x−1)(x−2)(x−3)
P.g. 187 !!! P (x) = 2 · (1−2)(1−3)(1−4)
+3· (2−3)(2−4)(2−1)
+4· (3−4)(3−1)(3−2)
+5· (4−1)(4−2)(4−3)
.
(2−3)(2−4)(2−1)
P.g. 188 !!! P (2) = 0 + 3 · (2−3)(2−4)(2−1)
+0 + 0 = 3.
P.g. 188 It can be viewed as a generalization of the facts that two points uniquely determine a straight line,
and intuitively follows since a degree n polynomial has n + 1 coefficients, and hence n + 1 degrees
of freedom.
P.g. 191 Comment 7.3.1: Alternatively, since a ≡ b (mod (a − b)), hence
7
7
Aditya Khurmi (Last Updated: June 8, 2021) MONT Typos and Corrections
P.g. 214 Using the Legendre’s symbol, we basically have converted the English question “is x a quadratic
residue” to a mathematical expression.
√
P.g. 214 !!! Prove that for a prime p > 3, the smallest quadratic nonresidue is smaller than p+1.
√
P.g. 214 !!! The idea is simple, pick the smallest quadratic nonresidue r, and try to show r < p (we
√ √
only consider p instead of p + 1 for now).
P.g. 215 !!! If we try to mend this idea, we look at numbers of the form r, 2r, . . . , (r − 1)r, r2 .
..
.
Hence, we get p + r > ra > p. Hence, ra mod p lies in {1, 2, . . . , r − 1}, which means it must be a
QR, a contradiction (if a 6= r)!
√ √
However, if a = r, then p + r > r2 implies r < 12 4p + 1 + 1 < p + 1, and so we are done in
this case too.
P.g. 215 “crosses” p
P.g. 215 We will see a generalization of this result in the chapter “Constructions.”
P.g. 215 Explain the significance of “reciprocity” in the theorem’s name.
P.g. 216 It is given as an exercise problem (with solution) in the chapter “Constructions”.
5−1 3−1 5−1
P.g. 217 (Latex error) (−1) 2 · (−1) 2 · 2 53
P.g. 218 Just recall that {an + b} forms a complete residue class mod p if gcd(a, p) = 1. So
P.g. 218 We need a more general method. So let’s try to find one.
P.g. 220 !!! Corollary 8.4.1. Let p be an odd prime and a, b be integers both coprime to p. Then
p
an2 + b
X a
=− .
i=0
p p
P.g. 220 Problem 8.5.1. Use the method from Example 8.1.1 to show that x2 + y 2 −1 ≡ 0 (mod p) always
has a solution x, y ∈ Fp .
P.g. 223 Clearly we can assume that gcd(x, y, z) = 1.
P.g. 224 !!! In particular, 5m − 1 is square free for primes > 2 (why?)
P.g. 255 is the trivial solution a1 = a2 = · · · = an = 0.
√ √
3
√
3
P.g. 256 The numbers {1, i} are independent, and so are {1, 3}. Here’s an exercise: Show that {1, 2, 4}
are independent.
P.g. 256 The classic proof of that is the “reductio-ad absurdum” method.
P.g. 257 Since surds modulo p keep changing on changing p, there is no direct way to do this.
P.g. 257 !!! Add conjuagte lemma to explain properly: “Also, the other roots of P (x) (the conjugates of
B) are also integer mod p by the lemma”
P.g. 259 Show that n = |Sp | = 1.
P.g. 266 !!! Fix i < j. How many values of k satisfy ai + ki ≡ aj + kj (mod p)?
P.g. 272 (change variable α to avoid confusion) 377. Write p + m − 1 = pβ and use pβ + m − 1 | n.
P.g. 280 Let s = 2α 5β γ, where γ is coprime to 10.
P.g. 281 So z1n ≡ z2n ≡ x (mod p) and z1n ≡ z2n ≡ y (mod p).
P.g. 282 Since n < p, hence p - n meaning that the right side of the above expression is non-zero, and
hence x1 + · · · + xn 6≡ 0 (mod p). Now the above holds for all a ∈ Sp , which is impossible since
a ≡ −n · (x1 + · · · + xn )−1 +1 (mod p) is unique.
√
P.g. 284 !!! However, we know ϕ(n) ≥ n for n 6= 2, 6. Now linear growth is faster than logarithmic
2 √
growth. Hence c(log n) < n for large enough n, and we have our contradiction.
P.g. 286 Solution 4.9.9 (EGMO 2013/4)
P.g. 289 If k = 4, then (x, y) = (1, 1) works and we get
P.g. 293 China TST 2 2016/4
8
8
Aditya Khurmi (Last Updated: June 8, 2021) MONT Typos and Corrections
and hence
1 1 1 1
νp (an+1 ) − νp (a1 ) ≤ n+ + ··· + + ··· + νp (C)
n 1 1 n−1
1 1 1 1 1 1 1 1
= 1 + + + · · · + + · · · + + + ··· + + ··· + νp (C)
n 2 2
| {z } |n {z n} 1 1 n − 1
2 n
1 1 1 1
= n · + n · + ··· + n · νp (C) = Hn νp (C).
n 1 2 n
P.g. 298 !!! But R(mc−2 · m) and R(mc−2 )R(m) have the same prime divisors, and since R(mc−2 ) > 1,
hence R(mc−1 ) has at least as many prime divisors as R(m). So, R(mc−1 ) and R(m) have the
same divisors.
k
So from every R(m2 ) we can induct down to prove the claim.
P.g. 299 Clearly we just have to show that 3 is a quadratic nonresidue.